MBE Criminal Law and Procedure Practice Problems

Ace your homework & exams now with Quizwiz!

The owner of a furnished cottage leased it to another for one year. While this lease was in effect, the cottage owner found herself in immediate need of cash, and decided to burn down the cottage to collect the insurance on it. She waited until one evening when the tenant was away. The cottage owner then used her own key to gain access to it. To make it appear that the fire was caused accidentally by the tenant, she soaked one end of the mattress on the bed in the bedroom with gasoline and then left a lighted cigarette burning at the other end of the mattress. She planned that the cigarette would ignite the mattress and that when the fire smoldered to the area soaked in gasoline, the entire bed would burst into flames, and the resulting fire would destroy the house. However, the tenant returned home earlier than expected and discovered the fire just as the mattress burst into flames. He immediately put it out with a fire extinguisher. A police investigation revealed the cottage owner's activities. The cottage owner is guilty of:

The cottage owner is guilty of burglary because the right of occupancy belonged to the tenant. However, the fact that there was no burning of the structure means that the cottage owner is guilty of attempted arson, rather than arson. Burglary at common law is a breaking and entering of the dwelling of another at nighttime, with the intent of committing a felony therein. A breaking requires some use of force to gain entry, but minimal force is sufficient. In determining whether the dwelling is that of another, occupancy rather than ownership is material. Thus, an owner can commit burglary of her own structure if it is rented and used as a dwelling by someone else. Here, although the cottage owner owned the cottage, the tenant had the right to occupy it pursuant to a lease. Thus, for purposes of the crime of burglary, the cottage owner is deemed to have entered the dwelling of another. Although the cottage owner used her own key to gain access to the cottage, this was still an unconsented use of force to effectuate entry, thereby constituting a breaking. This breaking and entering of the tenant's dwelling occurred in the evening. At the time of the entry, the cottage owner intended to commit the felony of arson. Consequently, all the elements of burglary are in place, making her guilty of this crime. Arson consists of the malicious burning of the dwelling of another. There is a requirement of some damage to the fiber of the wood or other combustible material. As with burglary, ownership of the structure is not material for determining whether the dwelling is that of another; rather, the right to occupancy is material. The cottage owner left a lighted cigarette on the mattress, intending to burn down the entire cottage. However, the tenant extinguished the fire before any damage was done to the structure of the cottage, even mere charring. Absent such damage, arson cannot have been committed. The cottage owner did commit attempted arson. A criminal attempt is an act which, although done with the intention of committing a crime, falls short of completing the crime. The defendant must intend to perform an act and obtain a result that, if achieved, would constitute a crime. Also, the defendant must have committed an act beyond mere preparation for the offense. The cottage owner intended to perform an act that would have culminated in the crime of arson. By soaking the mattress with gasoline and leaving a lighted cigarette on it, the cottage owner committed an act that came dangerously close to successfully burning the cottage. This act, in combination with the intent to commit arson, means that the cottage owner is guilty of attempted arson. (A) is incorrect because there can be no arson as to the mattress. Arson requires a burning of a dwelling. Because the cottage was not burned, the cottage owner is not guilty of arson. (B) is incorrect because the key element in determining whether a dwelling is that of another, for both arson and burglary, is the right of occupancy. Under the terms of his lease, the tenant had the right to occupy the cottage for one year. Therefore, the cottage owner's ownership of the cottage will not be a defense to either arson or burglary. (D) is incorrect because, as explained above, the cottage owner's use of a key to gain access to the cottage without the consent of the person who had the right of occupancy is deemed to be a use of force to gain entry, in the same way as if a person who did not own the cottage were to gain entry by means of a key.

A man and a woman were traveling in the man's car when they were stopped by the police for running a red light. Before the police came up to the car, the man told the woman, "You owe me a favor. Keep this package for me," and gave the woman a small foil package. The woman put the package in her backpack, saying, "O.K., but don't tell me what's in it." Before the police even began to question the occupants, the man blurted out, "I'm clean, man, but she has a stash," pointing at the woman. The officers searched the backpack that the woman was holding and found the foil package, which contained heroin. The woman was arrested, but the man was not. Is the evidence found on the woman admissible?

The evidence is admissible because the search was valid. Even though the police have validly stopped an automobile, they cannot search the vehicle without meeting the requirements of one of the exceptions to the warrant requirement, such as the automobile exception (which requires probable cause) or consent. The automobile exception comes into play when the police have probable cause to believe that the vehicle contains evidence of a crime. Under the exception, the police may search anywhere in the vehicle in which the item for which they have cause to search may be hidden, including packages in the vehicle. The statement of the man to the police officers gave them probable cause to believe that the car contained evidence of a crime (i.e., that the woman had drugs somewhere in the car). Thus, the requirement for application of the automobile exception was present, providing validity for the warrantless search conducted by the police. Because the search was valid, the evidence found on the woman is admissible. Besides being an incorrect statement of law, (B) is incorrect because it focuses on the woman's knowledge of the contents of the package. Whether the woman knew that heroin (or some other illegal substance) was in the package is irrelevant to the admissibility of the heroin. Even assuming that the woman knew of the contents, the search would not be valid unless there was a ground for the warrantless search. (D) similarly links the woman's knowledge of the package's contents to the admissibility of the evidence. As noted above, the admissibility of the evidence is dependent on the validity of the search that produced the evidence, rather than on the knowledge of the defendant as to the existence of the evidence. (C) is incorrect for three reasons: First, due process does not prohibit granting of immunity to a more culpable defendant. Second, there is no indication that immunity was even granted here (immunity from prosecution may be granted to compel a witness to answer questions. The facts merely state that the man was not arrested; this does not necessarily mean that he was granted immunity). Third, the call of the question relates to the admissibility of the evidence, and a grant of immunity does not relate to the question of the admissibility of the evidence found on the woman; such admissibility is determined by the validity of the search of the woman by the officers.

husband and wife were charged with stealing credit cards and charging expensive items on the misappropriated cards. An attorney was appointed by the court to represent the couple jointly. At the preliminary hearing, the judge found that the attorney would have no conflict representing both defendants in the joint trial. Halfway through the trial, however, a conflict arose between the defenses of the husband and wife. At the wife's request, the attorney moved that another attorney be appointed to represent the wife and that a mistrial be declared. The trial judge moved favorably on the attorney's motion. Another attorney was appointed to represent the wife, and as soon as the wife's trial began, her attorney moved to dismiss the case on the ground that jeopardy had attached during the wife's first trial and that she was being retried in violation of the United States Constitution. Should the judge grant the wife's attorney's motion?

Although jeopardy attached in the wife's first trial, her retrial is not barred because she initiated the grant of the mistrial in her first trial. As a general rule, the right to be free of double jeopardy for the same offense bars a retrial for the same offense once jeopardy has attached in the first trial. However, one of the exceptions permitting retrial even if jeopardy has attached is when a mistrial is granted in the first trial at the request of the defendant on any ground not constituting an acquittal on the merits. Here, the wife requested the mistrial because a conflict arose between the defenses of her and her co-defendant in the joint trial, and the judge granted the mistrial solely to allow the wife to obtain another attorney. Thus, no acquittal on the merits occurred and the double jeopardy rule does not apply. (A) is incorrect for several reasons. Merely because jeopardy attaches does not mean that the double jeopardy rule will apply; retrial will be permitted under certain exceptions, one of which is applicable here. Furthermore, (A) is not a correct statement of law. Jeopardy attaches in a jury trial when the jury is impaneled and sworn in, even if it has not yet heard any evidence. (B) is incorrect because the judge's finding at the preliminary hearing stage appears to be an honest error rather than bad faith conduct. In the absence of bad faith conduct by the judge or prosecutor designed to force the defendant to seek a mistrial, the defendant's securing of a mistrial does not preclude a retrial. (D) is incorrect because the right to be free of double jeopardy creates a bar as soon as the defendant is retried for the same offense, rather than on her conviction.

A bartender diligently followed the procedure her employer set: She would ask every patron for identification regardless of how old (or young) the patron appeared to be. One day, after asking for identification, the bartender served alcohol to a minor. The identification that the minor gave to the bartender was actually issued by mistake by an appropriate state agency and appeared to show that the minor was of legal age. After another patron, an off-duty police officer, recognized the minor, the bartender was arrested for serving alcohol to a minor. In this jurisdiction, the highest state court has held that, under state law, strict liability is abolished and all crimes require a culpable mental state. The best reason for finding the bartender not guilty would be:

An honest and reasonable mistake as to a material element of the offense would negate criminal liability for all crimes except strict liability offenses. Thus, if the state had abolished strict liability crimes, the bartender's mistake would be a defense regardless of the mental state required for the crime of serving alcohol to a minor. (B) is not as good an answer as (A). The bartender's lack of intent to commit the crime of serving alcohol to a minor would negate criminal liability if the crime required a specific intent, thus requiring an actual intention to engage in the act of serving alcohol to a minor. The question does not indicate the mental state required for the crime of serving alcohol to a minor. Even though the state had abolished strict liability offenses, the state could punish the crime of serving alcohol to a minor with a "reckless" or "should have known" state of mind. If so, the bartender's lack of intent would not result in a not guilty verdict. (C) is not as good an answer as (A) because the fact that the bartender made a diligent effort to determine the age of the minor would be an important consideration in deciding whether she made an honest and reasonable mistake, but it would not in and of itself automatically negate liability. A similar analysis applies to (D). The fact that the bartender checked the identification card supplied by a state agency would be an important consideration in deciding the nature of her mistake, but it would not by itself negate liability, as the mistake must be both honest and reasonable. For example, if the bartender knew that the minor was not of age despite what the identification card showed, she would commit a crime by serving the minor alcohol.

A defendant is charged with kidnapping a woman and killing her in a sacrificial ritual. Experts agree that the defendant suffers from schizophrenia. Under the M'Naghten rule, which of the following, if true, would entitle the defendant to an acquittal based on the insanity defense?

At the time of the crime, the defendant did not understand that his actions were actually occurring because he thought he was just watching them in a movie. Under the M'Naghten rule, a defendant is entitled to an acquittal if the proof establishes that a disease of the mind caused a defect of reason, such that the defendant lacked the ability at the time of his actions to either know the wrongfulness of his actions or understand the nature and quality of his actions. (C) meets this definition because the defendant did not understand the nature and quality of his actions when they occurred. He thought he was watching a movie, not actually committing the acts. (A) is wrong because a defendant is not entitled to an acquittal merely because he believes his acts are morally right, unless he has lost the capacity to recog- nize that they are regarded by society as wrong. Even though the defendant sincerely believed his victim's death was necessary, he recognized that society would punish him for his actions. (B) is wrong because, under the M'Naghten rule, loss of control because of mental illness is not a defense. (D) is wrong because even if the defendant's delusion had actually been correct, and the victim did consent to the ritual through telepathy, or otherwise, it would have still been a crime for the defendant to kill her.

A gang member determined that he needed to avenge a recent shooting of his friend by killing a member of the rival gang responsible for the shooting. He drank heavily to build up his courage and then went to t eh home of the rival with a loaded gun and knocked on the door. When the rival opened the door, the gang member pointed the gun at the rival and pulled the trigger. However, due to being intoxicated, he had forgotten to release the safety, so the gun did not discharge. He was easily disarmed by the rival and arrested by the police. At his trial, he testified that he was so intoxicated that he did not remember anything that happened at the rival's house. If the gang member's testimony is believed, what is the most serious crime of which the defendant may be convicted?

Attempted Murder. The most serious crime the gang member may be convicted of is attempted murder. At common law, murder is the killing of another human being with malice aforethought. Malice could be shown by the defendant's (i) intent to kill; (ii) intent to inflict great bodily injury; (iii) reckless indifference to an unjustifiably high risk to human life; or (iv) intent to commit a felony. Attempt requires the intent to commit the target crime plus an overt act in furtherance of such intent. Thus, even though murder is a malice crime at common law, attempted murder is a specific intent crime. As a result, voluntary intoxication, which is a defense to specific intent crimes, ordinarily will be a defense to any attempted murder charge. However, one who formed an intent to commit a crime and then drinks in order to work up his nerve to commit it cannot rely on the defense of intoxica- tion, even though he may be too intoxicated to form that intent at the time he did the act. Here, the gang member intended to kill his rival, went to his house with a gun, and pulled the trigger when his rival opened the door. Even if the jury believes his testimony that he did not remember anything that happened at the house, the fact that the defendant had the intent to kill originally and drank to build up his courage negates intoxication as a possible defense. Without such a defense, the defendant may be convicted of attempted murder. (B) is incorrect. Manslaughter requires either a killing committed during the heat of passion or a killing committed by criminal negligence. There are no facts indicating that the defendant was acting under the heat of passion. Furthermore, one cannot be convicted of attempted manslaughter based on negligence theory, because one logically cannot have the specific intent to be negligent. (C) is also incorrect. Although the defendant almost certainly put his rival in fear of an imminent unlawful battery or took sufficient steps to be guilty of the "attempted battery" type of assault, attempted murder would be the far more serious charge of which the defendant could be convicted. (D) is incor- rect because there are at least two crimes of which the defendant could be convicted—attempted murder and assault.

A police officer who stopped a driver for speeding noticed that the driver resembled the description of a hit and run river who had struck and killed a nine year old boy tow days before. The officer examined the driver's care an found recent dents. The driver was arrested for hit and run offense and read his Miranda rights. However, the officer failed to inform him that the child struck in the hit and run accident had subsequently died. On the way to the police station in the squad care, the driver blurred out, "You got me. I was the hit and run driver. I just hope the kid is ok." The officer still failed to inform him that the child had died. At the station house, the arresting officer said to the driver, "Look, you've already confessed. How about writing it down?" The driver replied, " I don't think I should write or sing anything without a lawyer, but I'll talk." He again admitted that he was the hit and run driver. The officer made a tape recording of his confession. In addition to speeding, the driving has been charged with leaving the scene of an accident and involuntary manslaughter. At the driver's trial, the prosecution seeks to admit both the first confession in the squad car and the second confession in the station house. The driver's attorney moves to suppress both confession.s How should the court rule on the motion?

Denied for both confessions. The driver's motion to suppress both confessions will be denied because the facts do not indicate a violation of his constitutional rights. Under basic principles of evidence, the state can introduce evidence of a confession on the part of the defendant in a criminal case. Such evidence would be considered an admission by a party. The defendant can have the confession suppressed if the confession resulted from a violation of the defendant's constitutional rights. On the facts presented in this question, the defendant might attempt to argue that: (i) Miranda was not complied with; (ii) the confession was involuntary; or (iii) the confession was the direct result of an illegal arrest. All three arguments will fail with respect to both confessions. Regarding the first confession, the driver's constitutional rights were not violated. (i) Miranda warnings are required prior to in-custody interrogation. Here, the defendant was not being interrogated; the facts state that he "blurted out" the confession. In addition, the warnings were, in fact, given. There is no require- ment under Miranda that the police inform the defendant of the possible charges that may be brought. (ii) The confession was voluntary. There is nothing in the facts to suggest coercion; rather, the facts state he "blurted out" the statement. (iii) The arrest was lawful; under the facts, the police had probable cause to arrest the driver. Regarding the second confession, (i) while the Miranda analysis is not as clear-cut, in all likelihood Miranda has not been violated. The warnings were given at the time of the arrest and, as this confession occurred within a short period of time in a conversation with the arresting officer, it is extremely unlikely that the court would require a second set of warnings. As to (ii), again the question contains no facts that suggest an involuntary confession, and (iii) shows that the police had probable cause to make the arrest. The driver may also claim that his right to have counsel present has been violated because he mentioned the need for an attorney. However, a request for counsel, accompanied by a willing- ness to speak without counsel, is a valid waiver of the right to have counsel present during an interrogation. Thus, the confession does not violate the driver's constitutional rights, and (A), (B), and (C) are incorrect.

Several students at a public high school told a teacher that a fellow student was selling illegal drugs to other students at the school. The accused student was called into the principal's office and informed of the accusations. The student denied everything, but he principal grabbed the student's purse, which was on top of his desk, and opened it. He removed five small transparent plastic bags, each of which contained a white powder, and immediately called the police. The police arrested the student and conducted tests confirming that the white powder was cocaine. The student was charged with possession of a controlled substance with intent to sell. At her trial, the state planned to introduce the bags and their contents into evidence. the student's attorney moved to suppress the evidence. How should the court rule?

Deny the motion, because the principal had a reasonable suspicion that the student was selling drugs. The motion should be denied. Due to the nature of the school environment, reasonable grounds for a search are a sufficient basis to justify searches by public school officials. Neither a warrant nor probable cause is required. A school search will be held reasonable if: (i) it offers a moderate chance of finding evidence of wrongdoing; (ii) the measures adopted to carry out the search are reasonably related to the objectives of the search; and (iii) the search is not excessively intru- sive in light of the age and sex of the student and the nature of the infraction. Here, even though the principal did not have probable cause to believe that the drugs were in the student's purse, the principal did have sufficient reasonable grounds to search her purse. Therefore, (A) and (B) are incorrect. (D) is incorrect because it is an overbroad statement of the rights of public school officials.

An irate woman seeking vengeance against an appliance store owner for selling her a defective television set wen tot he store armed with two pistols. Just as she was about to enter the store, she spotted a bond mother walking by with a baby carriage. The woman pointed one of the pistols at the mother and one at the carriage, saying, "I hate the owner of this store! Take this gun, go into his store, and do the job or the kid gets it!" The mother began pleading for her baby's life, but he woman cocked the pistol pointing at the carriage and began counting. The distraught mother took the other pistol and "did the job." Criminal charges are filed against the other. At trial, she asserts duress as a defense but is found guilty. What is "the job" she did no the story owner likely to have been?

Fatally wounding the owner with the pistol. The mother will be found guilty if she fatally wounded the owner with the pistol. The defense of duress requires the commission of an otherwise criminal act under the threat of imminent infliction of death or great bodily harm. Duress is a defense to all crimes except homicide. Note, though, that under certain circumstances it may reduce the homicide from murder to manslaughter. The crimes in (A), (B), and (D) would be offenses for which the mother could assert the defense of duress. However, even under duress, she would be found guilty of the homicide in choice (C).

A police officer outside a mall heard gunshots coming from the indoor dining area. He rushed into he dining area and saw a woman standing there with an assault rifle lying on the ground beside her. Several patrons were scattered prostrate about the dining area, some obviously suffering from gunshot wounds. The officer said, "What the hell happened here?" The woman replied in a dazed manner, "I just wiped out the lunch crowd." The woman was subsequently charged with several murders and attempted murders. At trial, testimony of the surviving customer was vague and conflicting. The prosecution offers the testimony of ht police officer, who will relate what he observed, including the woman's statement. Counsel or the woman objects to admission of any statements made by her to the police officer. How should the court rule?

For the state, because the woman was not in custody when she made the statement to the officer. The court should rule for the state. If a person questioned by the police has not been deprived of her freedom of movement in any significant way, she is not in custody and need not be advised of her constitutional rights. Thus, the rule of Miranda does not preclude evidentiary use of the woman's statement to the police officer. The fact that the woman was dazed does not change the admissibility of her statement. If she had been holding the assault rifle when the officer arrived, he probably would have had probable cause to suspect she had committed the offense, and at that time he would have had to read her the Miranda warnings. However, because the weapon was lying on the floor, the officer had no reason to suspect the woman of the crime. (B) is not the best answer because the problem is not the hearsay rule, because the woman's statement is clearly an admission. (C) is wrong because, as stated above, the woman need not be advised of her rights unless she had been taken into custody. (D) is wrong because the probative value of a confession is seldom outweighed by any unfair prejudicial effect. Answer choices that suggest this are almost always wrong.

To punish a gambler who owed him money, a bookie purchased from a demolitions expert a small explosive device that he planned to place under the driver's seat of the gambler's automobile. The demolition expert assured the book that the explosive would be strong enough to cause severe damages to the legs and ankles of anyone sitting in the front seat of the car, but would under no circumstances be strong enough to kill. The device would go off when the ignition was turned on. ON Sunday night, the bookie planted the device in the gambler's car. On Monday, the gambler was sick and did not go to work. That afternoon the gambler's 15 year old son came home from school and saw his father's care in the driveway. The son took an extra set of keys from the house to take the car for a joyride, even though the gambler had forbidden his son to drive the car without permission. When the 15 year old turned n the ignition, the explosive device went off and the boy died instantly. If the bookie is charged with the murder of the gambler's son what is the jury's verdict likely to be?

Guilty, because he intended to cause serious bodily harm to the gambler.The bookie is guilty because his act, which caused the death of the gambler's son, was committed with an intent to cause serious bodily harm. At common law, murder was the killing of a human being with malice aforethought. The mental state of malice aforethought could be established with any one of the following states of mind: (i) intent to kill; (ii) intent to cause serious bodily harm; (iii) reckless indifference to an unjustifiably high risk to human life ("depraved heart"); or (iv) the intent to commit a dangerous felony ("felony murder"). In this question, the bookie clearly intended to cause serious bodily harm to the gambler. Under the doctrine of transferred intent, if the actor intended to kill or cause serious bodily harm to one person and caused the death of another person, malice aforethought as to the unintended victim was established. Thus, under the doctrine, the bookie's intent to harm the gambler will be transferred to the gambler's son, and the bookie will be guilty of the murder of the gambler's son. (A) is not as good an answer as (B). It is true that the bookie's act caused the gambler's son's death and that cause must be established to hold the bookie responsible for murder. However, it will take more than "cause" to hold the bookie criminally liable; there must also be malice aforethought. Therefore, (A) is too broad a state- ment. (C) is wrong because, at common law, a person could be guilty of murder even if he did not intend to cause a death; intent to cause serious bodily harm, depraved heart, or felony murder are sufficient intent for malice aforethought. (D) is not a very good answer. The unforeseeability that the gambler's son would drive the car or get killed would not, in and of itself, be a defense. The unforeseeable result would at best be a factor in deciding whether the bookie's act was the criminal proximate cause of the death, but the facts in the question clearly establish proximate cause.

Three men agreed to rob a restaurant in a nearby town and bought pistols, ski masks, and gloves for that purpose. Prior to entering the restaurant, the man who was to be the lookout had a change of heart and wanted to call off the robbery. The other two men refused so the lookout threw down his gun and went to the nearby bus station to catch a bus back to his home. The remaining two men went into the restaurant and robbed it and it patrons. A patrol car happened by as they were leaving, and one of the men seized one of the customers as as hostage. In the exchange of gunfire with the police, a police officer and the hostage were killed. Both men escaped initially, but one of them was later captured and charged with robbery and murder. However, because of illegal police conduct in connection with his arrest and subsequent confession, all evidence connecting him with the charged crimes was suppressed and the charges dismissed. If the lookout is also arrested and charged with rude,r which of the following is his strongest defense?

He had withdrawn from the plan before the two others began the robbery that led to the killings. The lookout will not be guilty of murder if he withdrew from the plan before the robbery and killings took place. At common law, each person who took part in the planning of a crime was criminally liable for the crime of conspiracy and for each offense committed in furtherance of the conspiracy. However, if one of the conspirators "withdrew" from the criminal effort before the substantive crimes occurred, he was not liable for the subsequent crimes. To successfully withdraw, the actor must notify all members of the conspiracy that he has withdrawn; this must be done in time for them to have an opportunity to abandon the planned crimes. The facts in the question clearly indicate that he had withdrawn. (A) is not as good an answer as (C) because, if the lookout had not withdrawn, he would be guilty of murder under a felony murder theory. His strongest argument is that he withdrew. (B) is wrong because it is no defense to a charge of murder that the actor did not physically participate. The lookout would be guilty if he had not withdrawn, even without physical participation. (D) is wrong because, if the theory of the prosecutor's murder charge was that the lookout was a conspirator and is liable for all crimes committed in furtherance of the conspiracy, it would make no difference whether the other conspirators are being prosecuted. (All persons must be acquitted for this defense to be effective.)

A man and a woman were arrested and charged with conspiring to blow up a federal government building. After being given Miranda warnings, they were questioned separately and each of them gave a written confession. The confessions interlocked with each other, implicating both of the defendants as being involved in every stage of the conspiracy. Subsequently, the woman attempted to retract her confession, claiming that it was false. At a preliminary hearing, the judge rejected her claim. Both defendants were tried together, and the prosecutor introduced both confessions into evidence. At trial, the woman testified that she was not involved in any conspiracy and that her confession was fabricated. Both defendants were found guilty by the jury. If the woman challenges her conviction on appeal because of the admission of the man's confession and succeeds, it will be because:

If the woman prevails in her challenge to the admission of the man's confession it will be because the man could not be cross-examined regarding his confession. Under the Sixth Amendment, a defendant in a criminal prosecution has the right to confront adverse witnesses at trial. If two persons are tried together and one has given a confession that implicates the other, the right of confrontation generally prohibits the use of that statement because the other defendant cannot compel the confessing co-defendant to take the stand for cross-examination. A co-defendant's confession is inadmissible even when it interlocks with the defendant's own confession, which is admitted. If the man refused to take the stand and subject himself to cross-examination, his confession was not properly admitted because it violated the woman's Confrontation Clause rights. (A) is incorrect because the fact that the man's confession incriminates the woman more than her own confession is not relevant. Just the interlocking nature of the man's confession with the woman's confession makes it more damaging by making it harder for the woman to claim that her confession was false. (B) is incorrect because the Supreme Court has held that instructing the jury to consider the confession only as going to the guilt of the confessing defendant is inadequate to avoid Confrontation Clause problems, because the risk that the jury will not follow the limiting instructions is too great in this context. (D) is incorrect. Confessions of a co-defendant may be admitted if (i) all portions referring to the other defendant can be eliminated (so that there is no indication of that defendant's involvement), (ii) the confessing defendant takes the stand and subjects himself to cross-examination regarding the truth or falsity of the statement, or (iii) the confession of the nontestifying co-defendant is being used to rebut the defendant's claim that his confession was obtained coercively, and the jury is instructed as to that purpose. Even if the co-defendant denies ever having made the confession, as stated in choice (D), the opportunity at trial to cross-examine the co-defendant satisfies the Confrontation Clause.

A father who was paralyzed in an accident and unable to work refused to accept financial support from relatives or the government tholepin him care for his family. When one of his children became seriously ill and needed medical attention, the father refused to allow the child to receive medical care at the local public health clinic because of his feelings regarding the acceptance of charity. The father thought that the child was starting to get better, but then she took a turn for the worse and died from her illness. What crime has the father most likely committed?

Involuntary Manslaughter. The father has committed involuntary manslaughter. A father has a duty to provide the necessi- ties for his child. His failure to do so has caused the child's death. Such an omission is criminal, so (D) is incorrect. Criminal negligence, a form of involuntary manslaughter, is an unintended killing caused by the negligence of another. Criminal negligence requires a greater deviation from the reasonable person standard than is required for civil liability, but less negligence than the reckless disregard for human life required for malice. Certainly a parent's failure to provide medical treatment for a critically ill child is criminal negligence. Note that intent is not an element of involuntary manslaughter. (A) is incorrect because murder requires a showing of malice afore- thought, which includes a reckless indifference to an unjustifiably high risk to human life (i.e., an abandoned and malignant heart). Here the facts do not appear to rise to the level of culpability required to establish malice. (B) is not correct because this is not a heat of passion killing, which would be voluntary manslaughter.

A man asked a coworker who was a wine collector to lend him a bottle of expensive wine to put in his liquor cabinet, because he was inviting his mother over for dinner and wanted to impress her. The coworker permitted the man to take a bottle of wine worth $700 to his apartment for the dinner as long as he returned it the next morning. As he had planned all along, the man instead invited his girlfriend over for a romantic dinner, at which they drank the bottle of wine. The next day the man told his coworker that he had been mugged on his way home and that the muggers made off with he wine. Suspicious, the coworker found the empty wine bottle in the man's trash from his apartment? If the man is charged with theft in a common-law jurisdiction, of which theft offense is he most likely to be convicted?

Larceny by trick. he man can be convicted of larceny by trick. The owner of the wine gave possession of the bottle of wine to the man, but clearly did not give up ownership, because possession was transferred on the coworker's explicit promise to return it the next day. Therefore, a theft crime of some sort was committed, but that crime is not pure common law larceny because there was no trespassory taking. If the man obtained possession of the wine by fraud or misrepresentation, then he is guilty of larceny by trick. Because it apparently was the man's intent to drink the wine all along, the possession was obtained by fraud and the crime is larceny by trick, not embezzlement (the crime which would have occurred if he obtained rightful possession). Therefore, (D) is incorrect. (B) is incorrect because this is not a true trespassory taking; possession, even though obtained by fraud, was given. (C) is incorrect because, although the coworker had possession of the wine (which would normally indicate embezzlement), he obtained that possession by fraud (which indicates larceny by trick).

A felon planned to break in the rental storage unit next to his that contained valuable electronic equipment. He went to a hardware store to purchase a crowbar. The proprietor sold him the crowbar even though he told her that he needed it to break into someone's storage unit. After the purchase, the felon went to the storage facility with his friend. The felon told the friend that he had lost the key to his storage unit and did not have time to contact the facility's manager, so they needed to break into the unit to get his equipment. Because the felon had a bad back, the friend pried open the door with the crowbar and carried the equipment out to the car. A silent alarm was triggered and the pair were apprehended shortly after leaving the facility. Can the proprietor and the friend be convicted as accomplices to larceny?

No as to both the proprietor and the friend. Neither the proprietor nor the friend would be convicted as accomplices to larceny. An accom- plice is one who (i) with the intent to assist the principal and the intent that the principal commit the substantive offense (ii) actually aids, counsels, or encourages the principal before or during the commission of the crime. Here, the friend is not liable as an accomplice because nothing in the facts suggests that he disbelieved the felon's claim that it was his own storage unit. Hence, he had no intent to commit larceny, which requires an intent to permanently deprive another of his interest in the property. Therefore, (B) and (C) are incorrect. To be convicted as an accom- plice under the prevailing rule, a person must have acted with the intent to aid or encourage the principal in the commission of the crime charged. Absent a statute, most courts would hold that mere knowledge that a crime may result from the aid provided is insufficient for accomplice liability, at least where the aid involves the sale of ordinary goods at ordinary prices. Here, the proprietor's sale of the crowbar in the ordinary course of business would not make her liable as an accomplice under the prevailing rule even if she believed that the purchaser was going to use it to break into someone's storage unit. Hence, (A) and (C) are incorrect.

A homeowner decided to destroy his home by fire in order to collect the insurance. A neighbor's house was located a short distance form the homeowner's home. The homeowner knew that there was a strong wind blowing towards the neighbor's home; while he did not want to burn the neighbor's home, he nevertheless set fire to his own home. The fire department was unable to save the homeowner's house. They did manage to put out the fire moments before it spread to the neighbor's home, which suffered damage from smoke and soot. The jurisdiction's arson statute covers burning one's own dwelling as well as the dwelling of another, but is otherwise unchanged from the common law. If the homeowner is charged with attempted arson of the neighbor's home, is he likely to be found guilty?

No, because he did not intend to burn the neighbor's house. The homeowner will be found not guilty because he did not have the requisite mental state. To convict a person for an attempted crime, the prosecution must establish that the defendant had an actual specific intent to cause the harm prohibited by the statute and committed an act beyond mere preparation in furtherance of that intent. Those elements—specific intent and act—are required regardless of the mental state required by the target offense. A person who took a substantial step towards commission of the crime but was only reckless with respect to the target offense could not be found guilty of attempt. The homeowner did not intend to burn the neighbor's home. There- fore, he cannot be guilty of attempted arson of the neighbor's home. (B) is wrong. The fact that the fire was put out before it burned any of the neighbor's home would not preclude a conviction of attempted arson if the elements of attempted arson were otherwise established. (C) is wrong. To be guilty of attempted arson of the neighbor's home, the homeowner must have intended to burn the neighbor's home. The doctrine of transferred intent does not apply to attempt. (D) is wrong. A specific intent to burn the home is required for attempted arson. While malice satisfies the state of mind requirement for the completed crime of arson, it will not suffice for attempt.

Suspecting that a husband had slain his wife, police detective persuaded one of the husband's colleagues at work to remove a drinking glass from the husband's office so that it could be used for fingerprint comparisons with a knife found near the body. The fingerprints matched. The prosecutor announced that he would present comparisons and evidence to the grand jury. The husband's lawyer immediately field a motion to suppress the evidence of the fingerprint comparisons so as to bar its consideration by the grand jury, contending that the evidence was illegally acquired. Should the option be granted?

No, because motion based on the exclusionary rule are premature in grand jury proceedings. The motion should be denied. The issue in this question is not whether the seizure of the glass was valid, but whether it is an appropriate time to raise this issue. The exclusionary rule does not apply in grand jury proceedings. According to United States v. Calandra (1974), illegally seized evidence is admissible in grand jury proceedings. A pretrial motion to suppress is the appropriate vehicle to test the constitutionality of the seizure. The grand jury is not the appropriate forum. (A) is incorrect because the function of the grand jury is to consider evidence to determine whether there is probable cause to indict. It is not the function of the judge to take this matter away from the grand jury. A pretrial motion is the appropriate vehicle to challenge probable cause. (B) is incorrect because this choice goes to the legality of the seizure, which is not the issue in this question. The issue in this question is not whether the seizure of the glass was valid, but whether it is the appropriate time to raise the legality of the seizure. (D) is incorrect because it reaches the right result for the wrong reason. This choice goes to the legality of the seizure, which is not the issue in this question, as discussed in the explanation to (B).

A driver was stopped by the police after running a red light. Her roommate was also int eh car. Because the driver did not have a driver's license, the officer lawfully placed her under arrest and put her in his squad car. At the time of the arrest, the officer saw a shopping bag in the back seat containing clothes with price tags on them. The officer asked the driver if she had made any other purchases that day, and she responded that there were additional purchases in the trunk. The officer then searched the run of the car, where he found additional clothes purchases along with a clear plastic bag containing what appeared to be marijuana. Later testing confirmed that it was marijuana, which the roommates had purchased that morning form a neighbor. The two roommates were charged with possession of marijuana. Prior to her trial, the driver's attorney over to suppress evidence of the marijuana because it was discovered in an illegal search,a n the motion was granted. If the attorney for ht other roommate who was the passenger in the car subsequently moves to suppress evidence of the marijuana at her trial, should her motion be granted?

No, because she has no standing to object to an illegal search. The roommate's motion should be denied. The roommate has no standing to challenge the search of the trunk because the search did not violate her reasonable expectation of privacy. Merely being a passenger in someone else's car does not create a reasonable expectation of privacy with regard to a search of the car. Something more is needed to have standing to challenge the search, such as if the roommate owned the car (not indicated by the facts). Thus (A) is incorrect. (Note also that, while each passenger in a car stopped by the police has standing to challenge the stop because each person was seized along with the driver, the stop here was clearly proper: The police may validly stop a car for traffic violations, and here the driver ran a red light. Thus, the roommate's standing to challenge the stop is irrelevant.) (B) is incorrect because the judge's actions at the previous trial are irrelevant here. Whether the roommate has standing to challenge the search and seizure depends on her own reasonable expectation of privacy, regardless of whether the driver's rights were violated. (D) is incorrect. Essentially, it is an argument that, if the roommate admits that she owned the marijuana, she admits her guilt on the possessory charge, so her motion to suppress is irrelevant. However, at a suppression hearing, if a defendant charged with a posses- sory offense asserts a privacy (i.e., ownership) interest in the item seized, that testimony cannot be used against her at trial. [Simmons v. United States (1968)]

The sheriff's department received an anonymous tip that a farmer was growing marijuana on his rural property. Investigators flew low over the farmer's property in a small plane belonging to the sheriff's department and took aerial photos of the property. Once developed, the photos indicated that the area in the center of the farmer's fields contained marijuana plants. That afternoon, four officers went to the perimeter of the farmer's property. Using wire cutter, they cut their way through the farmer's barbed wire fence and walked to the center of the field and found the marijuana plants. The officers then obtained a warrant to search the farmer's house. ON arrival, they produced the search warrant and searched the farmer's house, finding large quantities of marijuana packaged and ready for sale. The marijuana was seized and the farmer was charged with numerous drug offenses. Prior to trial the farmer's attorney moves to suppress evidence of the marijuana seized from the house. Should the marijuana seized from the farmer's house be suppressed?

No, because the police and probably cause to obtain a warrant. The marijuana should not be suppressed. The search of the farmer's house, which led to the seizure of the marijuana, was based on probable cause and a valid warrant. Therefore, the evidence will not be suppressed. The United States Supreme Court has held that a person does not have a reasonable expectation of privacy in any land or field not a part of the curtilage. Thus, there is no Fourth Amendment protection in such areas. Therefore, the police did not violate the Fourth Amendment when they took pictures or when they cut through the fence and entered the field. All evidence obtained through those activities could be the basis for a warrant to search the farmer's house, and a search of a home based on a valid warrant will be considered reasonable under the Fourth Amendment. (A) is wrong because "open fields," even if they are fenced in, are not protected by the Fourth Amendment, and so a warrant to enter the lands was not necessary. The police may have committed a tort or perhaps violated the state criminal code when they cut the fence and entered onto the land, but the Fourth Amendment was not violated. (B) is wrong because the police could rely on anonymous information when deciding to investigate the farmer. The search of the house, which led to the marijuana, was based on a warrant and facts observed by the police. (D) is not as good an answer as (C) even though it is true that the open fields doctrine applies to the fields that the officers entered. Here, the motion to suppress relates to the marijuana taken from the house; for that, a warrant is required.

A man beat his live-in girlfriend and fled. The girlfriend called the police and told them about the beating. She also told them that the man likely feld to his best friend's house. The police quickly obtained a valid arrest warrant for the man and went to the friend's house a few hours after the beating. On arriving, the police noticed that a car registered to the man was parked nearby. They knocked and th friend answered the door. the friend told the police that the man was not there. The police pushed past the friend and began searching for the man. They found the man hiding in a closet and arrested him. On searching the man after his arrest, police found cocaine in a small metal box in the man's pants pocket. The man was charged with assault and possession of cocaine. In a pretrial motion, the man moved to suppress the cocaine, claiming that it was the fruit of an unconstitutional arrest. Should the court grant the motion?

No, because the police found the cocaine after executing a valid arrest warrant. The court should not grant the motion. The police may search a person after making a valid arrest. The arrest here was valid because the police had a valid arrest warrant. While the police should have obtained a search warrant to search for the man in the friend's home, the man does not have standing to complain of the illegal search. One may not raise a violation of another's constitutional rights at a Fourth Amendment suppression hearing. A person generally does not have standing to complain about a warrantless search of another's home unless the home was also his home or he was at least an overnight guest in the home. Here, the facts say that the man lived with his girlfriend, fled to the friend's house, and had been there a few hours. Thus, he lacks standing to complain about the warrantless search. (B) is incorrect. The community caretaker exception to the warrant requirement is not applicable here. The exception applies when police are acting to protect a person from imminent physical harm. Here, the beating had ended several hours before and the police had no reason to believe that anyone was in danger in the house. Therefore, the community caretaker exception does not apply. (C) is incorrect for the reasons stated above. While the police should have obtained a warrant to search the friend's home, the man lacks standing to complain about the warrantless search. (D) is incorrect because it is irrel- evant. Incident to arrest, police may perform a full search. Only a pat down conducted during an investigatory detention is limited to revealing weapons or items immediately recognizable as contraband.

Two men were arrested while riding in a stolen automobile. They were taken to the police station, booked, and fingerprinted. They were then taken to an interrogation room. After the detective gave them their Miranda warnings, one of the men said, "Forget it. AS soon as you check for outstanding warrants, you'll find out that I escaped from prison. Since I am going back anyway, it's a farce to deny that we stole that car." The other man said nothing , and the first man proceeding to write and sign a full confession. The man who remained silent pled not guilty to the charge of grand theft auto. At his trial the prosecutor seeks to introduce evidence to show that he did not deny that he stole the automobile when the other man told the police in from t of him that he was a party to the theft. Should the court hold that this evidence is proper?

No, because under this circumstance, the man who remained silent had no duty or responsibly to deny the allegations. The court should hold that this evidence is improper because the man who remained silent had no duty to deny the allegation here. A defendant in custody has no duty to speak at all, and the exercise of this constitutional right cannot be used against a defendant to show probable guilt. The Miranda warnings carry an implicit assurance that silence will carry no penalty. Thus, (A) is incorrect. (B) is immaterial, because the question only refers to the silent defendant's conduct and not the confessing defendant's statement. (C) suffers from the same problem, in addition to being a misstatement of the law.

A police officer patrolling at 2am heard a gunshot and saw a figure leap over a hedge in front of a condominium. Although it was dark, the officer could see that he was carrying a briefcase in one hand and a gun in the other. Just then, someone limped out of the building and pointed at the figure yelling, "He shot me!" The officer then twice commanded the figure to stop and find three warning shots in the air. The person kept running, so the officer fired a fourth shot, bringing the person down. The person was later identified as a robber. If the officer is charged with battery, what is the likely verdict of the jury?

Not guilty, because he had a right to use deadly force as necessary to stop a fleeing felon posing threat of serious bodily harm to others. The officer should be found not guilty of battery. Police officers are generally entitled to use whatever force is reasonably required, including deadly force, to apprehend or prevent the escape of a felon who poses a threat of serious bodily harm to the officer or others. Here, the robber had apparently just shot someone and was escaping with his gun, justifying the use of deadly force. (B) is an incorrect statement of law; there is no blanket immunity. (C) is wrong because an officer may use deadly force to prevent a getaway if the felon poses a significant threat of death or serious bodily injury to others. (D) is wrong because it is irrelevant whether the officer saw the robber steal anything. He saw the weapon and the wounded victim, and thus had probable cause to believe that the robber was dangerous.

A shopper at a flea market noticed a vase and asked a nearby person who much it cost. The person replied, "One hundred dollars." The shopper paid him the money and took the vase. The person, who was not the owner of the vase but merely a bystander, absconded with the $100. What crime at common law has the bystander committed with respect to the $100?

Obtaining property by false pretenses. The bystander has obtained property by false pretenses. In the crime of false pretenses, the defen- dant obtains title to the property by means of a false representation of a material present or past fact that causes the victim to pass title to his property to the defendant, who knows his representa- tions to be false and intends thereby to defraud the victim. Under the circumstances, by replying that the price of the vase was $100, the bystander implied that he was its owner or was authorized to sell it, which he was not. This false representation induced the victim to pass title to his money to the bystander, who knew the representation to be false. Therefore, he is guilty of obtaining money by false pretenses and (D) is incorrect. (A) is incorrect because the bystander took title, not mere possession. Therefore, the crime is false pretenses, not embezzlement. The physical transfer of that cash without any limitations on its use was a transfer of title to the money. Therefore, the crime is the obtaining of money by false pretenses. (B) is incorrect because larceny by trick occurs when the defendant obtains possession of another's property by lying or trickery. Here, the bystander obtained title to, rather than mere possession of, the money, and is therefore not guilty of larceny by trick.

A grand jury was investigating a bank robbery. The only information known to the prosecutor was a rumor that a certain ex-convict might have been involved. The grand jury subpoenaed the ex-convict. He refused to answer questions about the robbery and was granted use immunity. He then testified that he and a friend had robbed the bank. The grand jury indicted both the ex-convict and his friend for the bank robbery. The prosecutor permitted the friend to enter a plea to a lesser offense in exchange for the friend's agreement to testify against the ex-convict. The prosecutor had no evidence as to the identity of the robbers except the testimony of the friend and the ex-convict. At the ex-convict's trial, should his objection to his friend's being permitted to testify be sustained?

Testimony obtained by a promise of immunity is by definition coerced and therefore involuntary. Thus, immunized testimony may not be used for impeachment of the defendant's testimony at trial. The friend's testimony will not be permitted to be used against the ex-convict because it resulted from the ex-convict's immunized testimony. (A) is wrong because it is an inaccurate statement of the law. Prosecutors can bargain away the rights of co-defendants. (C) is wrong because police suspicion is not the equivalent of actual testimony. (D) is wrong. Even though a witness wants to testify, various privileges such as lawyer-client, doctor-patient, etc., may bar the testimony. Here, the grant of immunity to the ex-convict is a bar to the friend's derived testimony because use immunity bars use of one's testimony or anything derived from it.

A police officer was given a tip about a blonde male living in a nearby trailer park who was selling narcotics. The officer immediately drove to the trailer park and obtained from the manager the names of six blonde males who had trailers or mobile homes in the trailer park. At the first lot, the officer knocked on the defendant's door, announced that he was a police officer, and asked to talk to the defendant. The defendant's girlfriend, who did not live there but had been visiting, told the officer that the defendant would not be back for some time. The officer, believing that the girlfriend lived there, told her that he suspected that the defendant was dealing drugs and asked her if he could look around a little. The girlfriend said, "Sure, why not?" and let the officer in. After seeing nothing in the main living area, he went into the small back bedroom and opened several small storage compartments. In the corner of one of the compartments, he found an opaque bag. On opening it, he observed that it contained what appeared to be marijuana and confiscated the bag. Shortly thereafter, the defendant was arrested and charged with possession of narcotics with intent to distribute, a felony. On a motion by the defendant's attorney to suppress the introduction of the marijuana into evidence, how is the court likely to rule?

The court should deny the defendant's motion because the officer reasonably believed that the defendant's girlfriend lived in the trailer, making the search valid. Under the exclusionary rule, evidence obtained from an unconstitutional search must be excluded from trial. To be valid, searches must be reasonable. The Supreme Court has held that most searches are unreasonable unless the police obtain a warrant before searching. However, there are six categories of searches that the Court has held to be reasonable without a warrant. One such category is searches conducted pursuant to consent. To fall within this exception to the warrant requirement, consent must be given by one who appears to have an apparent right to use or occupy the premises and the search cannot go beyond the scope of the consent given. The consent is valid as long as the police reasonably believed that the person who gave the consent had the authority to do so, and the scope of the consent is limited only to areas to which a reasonable person under the circumstances would believe it extends. Here, the girlfriend's consent was valid because the officer believed that she lived there. His belief appears to be reasonable because she answered the door, knew of the defendant's whereabouts, and readily consented to the search. Therefore, the search was valid under the consent exception and the evidence should not be excluded. (A) is incorrect because consent is not invalid merely because the person who gave it did not actually have authority to do so; the police need only reasonably believe that the person had authority to consent, and as explained above, it was reasonable for the officer to believe that the defendant's girlfriend had authority here. (B) is incorrect because the scope of consent extends to any area where a reasonable person under the circumstances would assume it extends. Because the officer told the girlfriend that he suspected the defendant of dealing drugs, it was reasonable to assume that he was looking for drugs and so would probably look in even small containers. (C) is incorrect because it appears that the defendant's trailer would not fall within the automobile exception. Certain searches of automobiles are excluded from the requirement of a warrant because the Supreme Court has held that people have a lesser expectation of privacy in an automobile than in other areas and automobiles are likely to disappear before a warrant can be acquired. The automobile exception extends not only to cars, but also to other vehicles that are readily mobile and as to which there is a lesser expectation of privacy. However, nothing in the facts here indicates that the defendant's mobile home may readily be moved, and because it appears to be the defendant's regular home rather than a vehicle, it is doubtful that the Court would find the requisite lesser expectation of privacy. Therefore, the trailer would not fall within the automobile exception to the warrant requirement.

The police obtained a valid arrest warrant for a drug dealer. A reliable informant told the police that the drug dealer was staying at a friend's house until "the heat was off." Without having obtained a search warrant, the police went to the friend's house, knocked on the door, and asked the friend if the drug dealer was there. The friend replied that the drug dealer had been staying at the house for a few days but had left a few hours ago. The police pushed open the door and began searching for the drug dealer. They found him hiding in a closet along with two five-pound bricks of marijuana. They arrested both the drug dealer and the friend. Before his trial for possession of marijuana, the friend moved to suppress the marijuana found in the closet. Should the court grant the motion to suppress?

The court should grant the motion to suppress because a search warrant was required. Absent exigent circumstances, the police executing an arrest warrant may not search for the subject of the warrant in the home of a third party without first obtaining a separate search warrant for the home. If the police do execute an arrest warrant at the home of a third party without obtaining a search warrant for the home, the arrest is still valid but evidence of any crime found in the home cannot be used against the owner of the home because it is the fruit of an unconstitutional search. Thus, (A) is correct and (D) is incorrect. (B) is incorrect because it is too broad. A person can be arrested at the home of a third party, but the police generally cannot enter the third party's home without consent unless they have a search warrant for the home. (C) is incorrect because, as discussed above, a search warrant is required absent exigent circumstances, which are not present in this case. Here, the probable cause established by the informant's disclosure would have enabled the police to obtain a search warrant.

A woman was arrested, given Miranda warnings, and questioned about an armed robbery. After she asked to speak with an attorney, the police stopped questioning her about the robbery. Several hours later, the police gave the woman a fresh set of Miranda warnings and began to question her about a different robbery. She did not repeat her request for an attorney and instead made several incriminating statements about the robbery. At the woman's trial for the robbery for which she made incriminating statements, the prosecution seeks to have her statements introduced into evidence. If the woman's attorney objects on appropriate grounds, the court should:

The court should sustain the objection because the police did not honor the woman's request for an attorney. At any time prior to or during a custodial interrogation, the accused may invoke a Miranda (Fifth Amendment) right to counsel. If the accused invokes this right, all questioning must cease until the accused is provided with an attorney or initiates further questioning himself. Thus, the police questioning of the woman about the robbery was improper, and she can have her statements excluded. (A) is incorrect. After receiving Miranda warnings, if an accused invokes the right to remain silent, the police cannot badger the accused. However, courts have ruled that if the police scrupulously honor the request, they can rewarn the accused and later resume questioning, at least about a different crime. Here, however, the accused did not simply invoke the right to remain silent, but rather requested an attorney. After such a request, as indicated above, all questioning must cease. (B) is incorrect because the accused does not need to reassert the right to an attorney; all questioning must stop until the accused is provided an attorney or resumes the questioning herself. (D) is incorrect. It is stating the rule for impeachment - a confession obtained in violation of a defendant's Miranda rights but otherwise voluntary may be used against the defendant for purposes of impeachment, but there is no such rule for use of the confession for other purposes.

A police officer learned from a reliable informant that a major drug deal was about to take place at a local restaurant. The officer obtained a search warrant for the restaurant and arrived with other uniformed officers to search the premises. While conducting the search, the officer searched several of the customers. While searching one of the restaurant's regular customers, the officer felt an object in the customer's pocket and pulled out a container filled with heroin. The customer was arrested and later convicted of possession of heroin. A state statute permits officers executing a search warrant to search persons on the premises if the officers reasonably expect danger to themselves or a risk of disposal or concealment of anything described in the warrant. If the customer challenges his conviction on the ground that his Fourth Amendment rights were violated, will he be successful?

The customer will be successful. To be reasonable under the Fourth Amendment, most searches must be pursuant to a warrant. The warrant must describe with reasonable precision the place to be searched and the items to be seized. A search warrant does not authorize the police to search persons found on the premises who are not named in the warrant. In Ybarra v. Illinois (1979), a case based on similar facts, the Supreme Court held that "each patron of the tavern had an individual right to be free of unreasonable searches, and presence at a location subject to search does not negate the requirement of probable cause to search the person present." (A) is incorrect because the validity of the statute is not the primary issue. Even in the absence of a statute, the search of the customer by the officer violated the customer's Fourth Amendment rights. (C) is incorrect because, as discussed above, the search warrant did not override the customer's Fourth Amendment rights. While the police would be able to search a person discovered on the premises for whom they had probable cause to arrest, because the search would be incident to a lawful arrest, here they searched the customer prior to an arrest and without probable cause. (D) is irrelevant; if a search is unconstitutional, it does not matter that it was authorized by statute. To the extent that the statute authorizes a search in violation of the Fourth Amendment, it is unconstitutional.

Suspecting criminal activity, a police officer acting without a warrant peeked through a small opening in the shutters of an apartment. The officer observed the apartment's tenant and the defendant making methamphetamine. The officer immediately entered the apartment and arrested the tenant and the defendant, and he confiscated the ingredients for the methamphetamine, the tools used for methamphetamine production, and any completed methamphetamine for evidence. The search is later ruled invalid at a suppression hearing. May the defendant now claim that her Fourth Amendment rights have been violated by the seizure of the ingredients, tools, and methamphetamine from the apartment?

The defendant can claim a reasonable expectation of privacy for Fourth Amendment purposes if she was an overnight guest of the owner of the place searched. To raise a Fourth Amendment claim of an unreasonable search or seizure, a person must have a reasonable expectation of privacy with respect to the place searched or the item seized. It is not enough merely that someone has an expectation of privacy in the place searched. The Supreme Court has imposed a standing requirement so that a person can complain about an evidentiary search or seizure only if it violates her own reasonable expectations of privacy. The Court has held that a person has a reasonable expectation of privacy any time (i) she owned or had a right to possession of the place searched, (ii) the place searched was in fact her own home, whether or not she owned or had a right to possession of it, or (iii) she was an overnight guest of the owner of the place searched. Thus, the defendant would have standing to challenge the search of the tenant's apartment if she was an overnight guest of the tenant. (A) is incorrect because standing to raise a Fourth Amendment claim does not exist merely because a person will be harmed by introduction of evidence seized during an illegal search of a third person's property. The person must establish that her own reasonable expectation of privacy has been violated. (C) is wrong because the fact that the defendant was not the owner or occupier of the apartment does not preclude her from challenging the search. As discussed above, an overnight guest may also have a reasonable expectation of privacy in the premises for purposes of the Fourth Amendment. (D) is incorrect. Although the defendant may have standing to object to the seizure of items if she claims ownership of them, that is not the only basis for raising a Fourth Amendment claim; she will have standing to object to the search of the apartment under the circumstances in (B) regardless of whether she claims ownership of the methamphetamine.

he defendant, while visiting the victim, asked for permission to borrow the victim's car so he could drive to a convenience store to buy cigarettes. In fact, he intended to keep the car and sell it for cash. The victim agreed, and the defendant took the car and drove off. After thinking about it further, the defendant decided that it would be wrong to sell the victim's car, and headed back to the victim's house. On the way back, the car was destroyed in a collision through no fault of the defendant. May the defendant be convicted of larceny?

The defendant has committed larceny. The defendant's change of heart after taking the car will not provide him with a defense because it is irrelevant. Larceny requires the taking and carrying away of the tangible personal property of another by trespass. In the instant case, the larceny was committed at the time he took the victim's car with the intent to permanently deprive him of possession. (Note that the taking was trespassory because the defendant obtained possession by misrepresentation—larceny by trick.) Given that the crime was complete once the defendant drove off, he is guilty of larceny, which makes (D) the correct answer. (A) is incorrect because the defendant did not intend to return the car at the time of the taking and thus had completed the crime at that time. He had the requisite mens rea at the time of the taking and carrying away, and this is sufficient to convict. (B) is incorrect because abandonment may be a defense in some states to an attempt of a crime. In the instant case, the defendant had already completed the crime, as described above. Thus, abandonment cannot act as a good defense. Choice (C) is incorrect because withdrawal is not relevant to the crime of larceny.

The defendant, who formerly worked for a construction company, became intoxicated one night and decided to move some heavy construction equipment that was parked at a construction site. Ignoring "no trespassing" signs, the defendant jumped the fence and climbed into a large dump truck and started it up. However, due to his intoxication, he quickly lost control of the truck. It rumbled a short distance and crashed into a trailer housing the main office of the construction site. The defendant is prosecuted for recklessly damaging property. The defendant should be found:

The defendant should be convicted because he was intoxicated when he damaged the trailer. The defendant is being charged with reckless damage to property. A person acts recklessly when he consciously disregards a substantial and unjustifiable risk that a prohibited result will follow, and this disregard constitutes a gross deviation from the standard of reasonable care. Attempting to move a large piece of construction equipment while intoxicated should be considered to be reckless conduct because of the great potential for destruction arising from the size and destructive power of the construction equipment. Therefore, (B) is correct. (A) is incorrect because merely driving the equipment in violation of the law would not necessarily be reckless. For instance, here, the statute likely was enacted to prevent untrained persons from driving dangerous equipment, but the defendant was trained to operate the truck in question; thus, if not for the fact that he was drunk, his action would not necessarily have been reckless. Violating the statute may be evidence of negligence, but negligence is insufficient to establish recklessness. (C) is incorrect for the same reason that (B) is correct—driving the equipment while intoxicated constitutes reckless conduct. Although voluntary intoxication is a defense to a crime that requires purpose or knowledge, it is no defense to crimes involving recklessness. Even though the defendant's condition may in fact have precluded him from being consciously aware of the risk, one who is not consciously aware of a risk only because he was voluntarily intoxicated will be deemed to have acted recklessly with regard to the risk. (D) is incorrect because it states the mental state for knowing conduct—if the defendant is aware that his conduct will necessarily or very likely cause a certain result, he acts knowingly with respect to that result. Recklessness is a lesser standard of fault.

The defendant was at a bar with a couple of friends when he spotted a man who had gotten the defendant's friend fired from a job several weeks ago. Since that time, the defendant had been verbally harassing the man and calling him names. This particular night, the defendant went over to the man's table and flirted with his girlfriend. The man was infuriated after having taken the defendant's abuse for so long, so he jumped up and attacked the defendant with a knife. The defendant could have easily run away, but instead grabbed the man and slammed him backwards. The man went crashing through the front window and was severely cut by the broken glass. He died before he could be taken to the hospital. The defendant would most likely be guilty of:

The defendant would most likely be guilty of none of the listed crimes because the defense of self-defense makes his homicide excusable. A person may use deadly force in self-defense if (i) he is without fault, (ii) he is confronted with unlawful force, and (iii) he reasonably believes that he is threatened with imminent death or great bodily harm. In a majority of states, a person may use deadly force in self-defense even if this could be avoided by retreating. Here, the defendant's use of force was privileged because it reasonably appeared necessary to defend him from the man's unlawful attack, and the defendant had no duty to retreat under the majority view. Furthermore, the defendant can claim the privilege of self-defense even though his words triggered the fight—calling someone names would not be considered adequate provocation that would make the defendant the aggressor. Hence, because the defendant's use of force was privileged, he cannot be convicted of any of the listed crimes, making (A), (B), and (C) incorrect.

A driver was operating her car on a city street when she was stopped by a police officer for speeding. As the police officer reached the driver's car, he saw her put something into her purse. The officer told the driver, "Ma'am, you were speeding; that's why I stopped you. I'd like your driver's license, and, by the way, what did you just put into your purse?" The driver responded, "It's just a marijuana cigarette, but don't worry, I've only had two and my driving judgment hasn't been impaired." The officer took her purse, removed the "joint," and charged the driver with possession of marijuana as well as speeding. At the driver's trial for marijuana possession, the prosecution seeks to introduce the marijuana cigarette into evidence. The driver's attorney moves to suppress the evidence. The defense motion should be:

The defense motion should be denied because the driver was not in custody when she made the statement. Persons temporarily detained for routine traffic stops are not in custody for Miranda purposes. Therefore, the driver was not entitled to Miranda warnings, and her statement about the marijuana was not tainted. Her statement thus properly provided the probable cause for the search of her purse. (A) is therefore wrong. (B) is wrong because this case falls within the automobile exception to the warrant requirement. (D) states the test for a stop, not a search. An automobile search requires probable cause.

The defendant, angered because a rival gang member had twice beaten him up after school, obtained a heavy lead pipe and waited in a deserted alleyway which he knew the rival took as a route home every day after school. When his enemy came walking down the alley, the defendant leapt out behind him and smashed the pipe into the victim's head, knocking him to the ground. The defendant then rolled the victim over and pounded his face with 15 to 20 heavy blows with the lead pipe, killing him. The jurisdiction defines first degree murder as murder committed with premeditation and deliberation. All other murders are defined as second degree murders. If the defendant is convicted of first degree murder (as opposed to second degree murder), it will be because:

The degree of murder under the statute is determined by the defendant's mental state--whether the killing was intentional and accomplished after premeditation and deliberation. The defendant's relationship with the victim and the manner of killing may have evidentiary significance with regard to the defendant's mental state, but do not themselves distinguish first from second degree murder. Thus, (A) and (C) are incorrect. The causal relationship between the defendant's act and the death of the victim may determine whether the act is murder, but once that analytical hurdle has been passed, it has no further significance as to the degree of murder. Therefore, (B) is incorrect.

A felon intending to rob a market waited outside until there were no customers. When he saw that the market was empty, he went inside and walked up to the counter with his hand in his jacket pocket to simulate a gun. Before the clerk could turn around to see what the felon wanted, another customer entered the market, startling the felon, who turned and ran out the door. On a charge of attempted robbery, the felon should be found:

The felon should be found guilty of attempted robbery. With the specific intent to commit a robbery, the felon went beyond mere preparation for the offense, and having done so, abandonment is not a defense. A criminal attempt is an act that, although done with the intention of committing a crime, falls short of completing the crime. The defendant must have the intent to perform an act and obtain a result that, if achieved, would constitute a crime. Also, the defendant must have committed an act beyond mere preparation for the offense. If a defendant has, with the required intent, gone beyond preparation, the general rule is that abandonment is not a defense. Even in those jurisdictions in which abandonment is a defense, such abandonment must be: (i) fully voluntary and not made because of the difficulty of completing the crime or because of an increased risk of apprehension; and (ii) a complete abandonment of the plan made under circumstances manifesting a renunciation of criminal purpose, not just a decision to postpone committing it or to find another victim. Here, the felon intended to take money from the clerk at the market by means of the threat of having a gun (i.e., by simulating a gun). Thus, the felon intended to commit a robbery. In walking up to the market counter while simulating a gun with his hand, the felon committed an act that was a substantial step toward commission of the intended crime, and that strongly corroborated his intent and purpose to commit the crime. All that was missing to complete the crime was for the clerk to turn around and, upon seeing the felon apparently armed, be forced to give up the money. Thus, the felon went far beyond mere preparation for the crime of robbery. Having gone beyond mere preparation, with the intent to commit robbery, the felon is guilty of attempted robbery. And, as explained above, even if the felon abandoned his plan when the customer entered the market, such abandonment will not afford him a defense. Even in those jurisdictions in which abandonment is a defense, the felon will not have a defense because his abandonment apparently occurred when the customer's sudden presence increased the risk of apprehension. Thus, the abandonment did not really manifest a renunciation of criminal purpose. (A) is incorrect because, to be guilty of attempted robbery, events need not have progressed to the point where the defendant has used or threatened to use force. Because the felon had the requisite intent for attempt and went beyond mere preparation by standing at the counter and simulating possession of a gun, he should be found guilty of attempted robbery. (B) is incorrect because, as detailed above, abandonment of an attempt does not afford a defense, and in any event, the felon's abandonment here did not really come about by way of renouncing his criminal purpose. (C) is incorrect because a conviction of attempt does not require entry into a "zone of perpetration." Rather, a defendant (with the requisite intent) need only have committed an act beyond mere preparation. The Model Penal Code and most state criminal codes require that the act constitute a substantial step toward commission of the crime and strongly corroborate the actor's criminal purpose.

Two robbers planned to commit armed robberies targeting older victims. However, when the time came to actually commit the robbery, one of the robbers, thinking that the potential victim looked too much like his grandmother, backed out and told his cohort that he was going home. The second robber went ahead with the plan and robbed the elderly victim, who died of a heart attack due to the stress of the robbery. The second robber was arrested and implicated the first robber.

The first robber is guilty of conspiracy but not murder. The conspiracy was complete when the robbers agreed to commit the robbery and targeted their first victim. The first robber's withdrawal is no defense to the conspiracy charge. Hence, (A) is incorrect. The first robber is not guilty of murder, however, because of his withdrawal. The murder charge would be based on felony murder, because the second robber caused the foreseeable death of the elderly victim from the heart attack during the commission of the felony. However, a conspirator may limit his liability for subsequent acts of other conspirators by performing an affirmative act that notifies the other members of the conspiracy. Here, the first robber told the second robber that he was going home in time for the second robber to abandon his plans. Hence, he is not liable for felony murder arising from the robbery, making (C) and (D) incorrect.

A husband discovered his wife in bed with a neighbor. The neighbor ran out the back door. The husband screamed at his wife and vowed revenge. After consuming several drinks to build up his nerve and becoming intoxicated, the husband went to his friend's house and borrowed a gun, and then went to the neighbor's house. The neighbor had neglected to lock his front door, so the husband walked in. He found the neighbor trembling in the living room and pointed the gun at him. The neighbor immediately began apologizing and pleading for his life, but then suddenly he pulled a switchblade knife from his pocket. As the metal flashed, the husband fired a single shot at the neighbor, killing him. What is the most serious crime of which the husband can be convicted?

The husband can be convicted of murder. Murder is the unlawful killing of another human being with malice aforethought, which may be (i) intent to kill, (ii) intent to inflict great bodily injury, (iii) reckless indifference to an unjustifiably high risk to human life, or (iv) intent to commit a felony. Intentional use of a deadly weapon authorizes a permissive inference of intent to kill. Here, the husband uttered statements of revenge, confronted the neighbor with a loaded gun, and intentionally shot him when he pulled out a knife—more than enough evidence for a jury to find that the husband had the malice aforethought necessary for murder. Furthermore, none of the issues raised in the other choices will suffice to excuse the killing or reduce it to voluntary manslaughter. (B) is incorrect because the husband will not be able to meet all four tests for establishing the provocation necessary to reduce a killing from murder to voluntary manslaughter. The husband would have to offer evidence that (i) a provocation existed that would arouse sudden and intense passion in the mind of an ordinary person such as to cause him to lose his self-control, (ii) the husband was in fact provoked and lost his self-control, (iii) there was not sufficient time between the provocation and the killing for the passions of a reasonable person to cool, and (iv) the husband in fact did not cool off between the provocation and the killing. The husband can easily establish the first two elements, because discovery of one's spouse in bed with another person is virtually always considered adequate provocation by common law courts. However, the time interval between the provocation and the killing was probably sufficient for a reasonable person to cool off, and the facts strongly suggest that the husband did in fact cool off—he consumed several drinks to build up his nerve and went to a friend's house to get a gun before confronting the neighbor. Thus, a jury would probably reject a claim of voluntary manslaughter here. (C) is incorrect because the husband's voluntary intoxication would not preclude a finding of intent for murder. Because the husband became intoxicated to build up his nerve to kill the neighbor, a court would probably find that his intent at the time he began drinking would apply to his later conduct. Furthermore, voluntary intoxication is no defense to crimes involving recklessness. The husband can still be liable for murder based on a state of mind of reckless indifference to human life—his conduct in becoming intoxicated and then confronting the neighbor with a loaded gun is sufficient to establish that state of mind. (D) is incorrect because the homicide will not be excused on self-defense grounds. A person may use deadly force in self-defense only if (i) he is without fault, (ii) he is confronted with unlawful force, and (iii) he is threatened with imminent death or great bodily harm. The husband is not without fault, however, because he initiated the assault and prompted the neighbor to pull the knife. His status as the aggressor deprives him of the right to use force in his own defense under these circumstances.

Two robbers planned to rob a local convenience store, with one using a gun to force the clerk to turn over all of the money in the cash register while the other stood lookout near the door. The robbery did not go as planned. Instead of turning over any cash, the store clerk tried to disarm the gunman. During their struggle for the gun, the lookout decided that her best course of action was to grab what she could and flee the scene. The lookout took a newspaper and a bag of potato chips and ran out of the store. On her way out, she heard a gunshot. Later that day, she learned from news accounts that the gun accidentally discharged, killing the gunman. After an investigation, the lookout was arrested. If the lookout is charged with felony murder, her most promising defense would be:

The lookout's best defense is that the gunman was the only person killed. Under the felony murder doctrine, a killing committed during the course of a felony is murder, malice being implied from the intent to commit the underlying felony. However, under the majority view, criminal liability for murder cannot be based on the death of a co-felon from resistance by the victim or police pursuit. Thus, given that the gunman's death resulted from an act by the clerk, the victim of the robbery, the lookout cannot be found guilty of the felony murder of the gunman, a co-felon. (A) is incorrect because any desire or lack of desire by the lookout to see her co-felon harmed is irrelevant to liability for felony murder. The only mens rea required is the intent to commit the underlying felony. Here, the lookout had the intent to commit robbery, the underlying felony. From this intent, the malice required for murder is implied. (C) is incorrect because the fact that the felony was technically completed before the gunman's death does not prevent the killing from being felony murder. A death caused while fleeing from the crime is considered to have been caused during the commission of the felony. (D) is incorrect because robbery is a felony regardless of the value of the property that is taken.

cting with probable cause, the police arrested a man in connection with the armed robbery of a liquor store. After being given Miranda warnings, the man confessed to the robbery but denied his involvement with several other recent armed robberies of businesses in the area. He was formally charged with the one robbery and put into a cell with a paid informant working undercover for the police. The informant had been instructed to find out what he could about the other robberies but not to ask any questions. The informant began talking about a convenience store robbery in which a bystander was shot and seriously injured by the robber, and he deliberately misstated how it happened. The man, unaware that his cell mate was an informant, interrupted to correct him, bragging that he knew what really happened because he was there, and proceeded to make incriminating statements about the robbery. The man was subsequently charged with armed robbery and attempted murder in the convenience store robbery. At a motion-to-suppress hearing on that charge, if the man's attorney moves to exclude the statements made to the informant, should the motion be granted?

The man's motion should be denied because neither his Fifth nor Sixth Amendment rights were violated by the informant's conduct. The Sixth Amendment right to counsel applies to all critical stages of a criminal prosecution after formal proceedings have begun, but does not apply in precharge custodial interrogations. Because this right is "offense specific," the fact that the right to counsel has attached for one charge does not bar questioning without counsel for an unrelated charge. Because the man has not been charged with the convenience store robbery, his Sixth Amendment right to counsel has not been violated. The Fifth Amendment privilege against self-incrimination requires Miranda warnings and a valid waiver before any statement made by the accused during custodial interrogation can be admitted. However, this requirement does not apply where interrogation is by an informant who the defendant does not know is working for the police, because the coercive atmosphere of police-dominated interrogation is not present. [Illinois v. Perkins (1990)] Because the man was not aware of the informant's status, the informant's conduct did not constitute a police interrogation. (A) is wrong despite the fact that the informant's conduct may have been deliberately designed to elicit incriminating remarks. As discussed above, the man's right to counsel did not attach for purposes of the convenience store robbery. (B) is incorrect because, as discussed above, the Miranda warnings need not be given before questioning by a cellmate working covertly for the police. (D) is incorrect because interrogation refers not only to express questioning, but also to any words or actions on the part of the police that the police should know are reasonably likely to elicit an incriminating response from the suspect. Here, the informant, working for the police, made statements about the convenience store robbery that were intended to, and reasonably likely to, prompt a response from his cellmate. Hence, it is not the absence of "interrogation" that avoids the Miranda problem, but the fact that the man did not know that his cellmate was working for the police.

A drug addict entered a pawnshop with a starter pistol that could not fire real bullets, intending to rob it so he could buy drugs. However, once inside the pawnshop, the addict was too afraid to do anything and turned to leave. He bumped into another customer who was coming in as he was going out, and the pistol fell from his coat pocket. The shopkeeper saw it and realized what the addict had intended. She chased after him and flagged down a patrol car, and he was arrested a block away. The addict is charged with burglary, which is defined in this jurisdiction as "breaking and entering of any building for the purpose of committing a felony. His best defense would be which of the following?

The pawnshop was open for business. Because this statute requires that the defendant "break and enter," the addict's best defense is that, because the pawnshop was open to the public, he had not "broken" in but entered with the implied consent of the pawnshop owner. (A) is wrong because it is immaterial to the crime of robbery (the underlying felony in this burglary) that the robber was incapable of carrying out his threat of harm, as long as the victim reasonably believed that he was. (C) is not the best answer because the statute requires only that the defendant enter the building with the intent to commit a felony. The fact that the intent was abandoned after entry would not make the defendant not guilty of burglary. (D) is wrong because there is nothing in the facts indicating that the addict was incapable of realizing what he was doing, and being a drug addict in and of itself does not show that the defendant is incapable of forming an intent to commit a crime.

A college student was the sole lifetime beneficiary under a large trust administered by a banker. The student received a large monthly distribution from the trust, and whenever he ran short, he simply called the banker for extra funds, because the trust provided that the student was to receive whatever he needed from income or principal. The student's roommate found out about the trust arrangement and decided to see if he could make it pay off for him. The roommate sent a telegram to the banker, which appeared to be from the student, and which asked for several thousand dollars to cover medical expenses. The telegram further stated that, since he was in the hospital, the student would send his roommate to pick up the cash. The next day, the roommate showed up at the banker's office and obtained the money on the promise that he would take it to the student. The roommate absconded with the funds. When the roommate obtained the cash from the banker, he committed:

The roommate committed larceny by trick because the banker's consent to the roommate's taking the money was induced by the misrepresentation that the roommate would take the money to the student/beneficiary. Larceny consists of a taking and carrying away of tangible personal property of another by trespass, with intent to permanently (or for an unreasonable time) deprive the person of his interest in the property. If the person in possession of property has not consented to the taking of it by the defendant, the taking is trespassory. However, if the victim consents to the defendant's taking possession of the property, but such consent has been induced by a misrepresentation, the consent is not valid. Under such circumstances, the larceny is called larceny by trick. Here, the roommate obtained the money from the banker on the promise that he would take it to the student/beneficiary. This misrepresentation induced the banker to give possession of the money to the roommate. The roommate then proceeded to take the money and carry it away, intending all the while to permanently deprive one who had a possessory interest superior to the roommate's of his interest in the money. Thus, all the elements of larceny are present. Because the original wrongful taking resulted from consent induced by misrepresentation, the specific larceny committed by the roommate is more precisely characterized as larceny by trick. Consequently, although the roommate has in fact committed larceny, (C) is a better answer than (D). Regarding (A), false pretenses consists of obtaining title to the property of another by an intentional (or knowing) false statement of past or existing fact, with intent to defraud the other. If a victim intends to convey only possession of the property to the defendant, the offense is larceny by trick. However, if the victim intends to convey title, the offense is false pretenses. Here, the banker intended to convey possession of the money to the roommate so that he could give the money to the student/beneficiary. The banker did not intend to convey title to the roommate. Because the roommate did not obtain title by means of his misrepresentation but simply obtained possession, the offense of false pretenses was not committed. (B) is incorrect because embezzlement is the fraudulent conversion of property of another by a person in lawful possession of that property. In embezzlement, the misappropriation of the property occurs while the defendant has lawful possession of it. In larceny, the misappropriation occurs generally at the time the defendant obtains wrongful possession of the property. The roommate did not have lawful possession of the money because his possession of the money resulted from his misrepresentation to the banker. Thus, the roommate's taking of the money was wrongful from the outset. Because the roommate had wrongful, rather than lawful, possession of the money, there was no embezzlement.

The defendant and an accomplice were committing an armed robbery of a convenience store when the police arrived. A shootout ensued, resulting in the death of someone on the scene. The defendant is arrested and charged with felony murder in a jurisdiction that follows the agency theory of liability for felony murder. The defendant is most likely to be found guilty if the victim was which of the following?

The store clerk shot by the defendant's accomplice after the clerk had pulled out a gun from under the counter. The defendant will be most likely guilty of felony murder for the shooting by his accomplice. Under the felony murder doctrine, a killing committed during the course of a felony is murder. Under conspiracy and accomplice liability law, all participants in the felony will be liable for murder if the killing was foreseeable. Resistance by the store clerk was foreseeable and neither the accomplice nor the defendant had any right of self-defense under the circumstances. Hence, the defendant is liable for the shooting of the store clerk by his accomplice during the armed robbery. (B) is incorrect because, in states following the agency theory of felony murder, the killing must have been caused by the defendant or someone acting as the defendant's "agent" (i.e., an accom- plice). Thus, the killing by the police would not result in the defendant's liability for felony murder. (C) is incorrect because the majority (Redline) view is that liability for felony murder cannot be based on the death of a co-felon from resistance by the victim or police pursuit. (D) is incorrect for the reason discussed in (B), above.

The police of a resort town discovered that a well-known cat burglar was currently living in town under an assumed name. To try to catch her in the act of burglary, an undercover officer approached the burglar with a plan for a burglary. The undercover officer told the burglar that he knew who she was and that he had a plan to steal jewels from someone staying in one of the town's resorts. The burglar initially refused the offer; however, after lengthy cajoling, she finally agreed to the plan. As the time for the burglary drew near, the burglar had second thoughts. Three hours before the theft was scheduled to take place, the burglar called the police and told them of the plan. She told them that she was not going to show up, but that her cohort (the undercover officer) would be there, and told them how to recognize the undercover officer. Is the burglar guilty of conspiracy at common law?

There was an insufficient agreement for conspiracy liability at common law. Conspiracy consists of: (i) an agreement between two or more persons; (ii) an intent to enter into an agreement; and (iii) an intent to achieve the objective of the agreement. In addition, most states require an act in furtherance of the conspiracy, although an act of mere preparation will usually suffice. The agreement requirement means that the parties must agree to accomplish the same objective by mutual action. There must be a meeting of at least two "guilty minds"; i.e., between two or more persons who are actually committing themselves to the scheme. If one person in a two-party conspiracy is only feigning agreement, the other person cannot be convicted of conspiracy under the common law bilateral approach. Here, the officer, in his undercover capacity, was simply trying to set up a situation in which the burglar would be caught. Thus, the undercover officer merely pretended to reach an agreement with the burglar to commit a burglary. At no time did the undercover officer actually commit himself to the burglary. Therefore, there could have been no agreement of two "guilty minds." Absent the requisite agreement, the burglar cannot be guilty of conspiracy. (A) is incorrect because, as explained above, there was no agreement sufficient for a conspiracy conviction, since the undercover officer never intended to commit the burglary. (C) is incorrect because completion of the substantive crime is not necessary for a conviction of conspiracy. Consequently, although the actual burglary was not consummated, this would not preclude a conviction of conspiracy to commit burglary. (D) is incorrect because withdrawal is not a defense to a charge of conspiracy. Note that, by withdrawing, a person may limit her liability for subsequent acts of the other members of the conspiracy. However, this question pertains to the burglar's potential guilt for conspiracy. As applied to the conspiracy charge, withdrawal will not afford a defense to the burglar.

One night when a man was very drunk, he took one of his rifles, loaded it, and fired a bullet through his front door. Unbeknownst to him, at the time he fired the rifle, someone was driving by the house. The bullet went through the front door, through the window of the car, and killed the driver. The shooter was convicted of murder and appeals. He contends that there was insufficient evidence to support a finding of murder. The court of appeals should rule that the evidence is:

Under the facts of this case, to support a finding of murder, the trial court would have to find that the shooter acted either intentionally or with malice aforethought. The facts clearly indicate that the shooter did not know of the car, so it cannot be said that he shot at it intentionally, and therefore (A) is not correct. "Malice aforethought" can mean that the defendant is acting with a "wanton" state of mind. There is little question that shooting a rifle through a front door can be considered "wanton." Thus, the question is whether the shooter's intoxication was sufficient to negate this state of mind. If a defendant's lack of awareness results from voluntary intoxication, his conduct will nevertheless be deemed wanton. (C) is not a correct analysis of the issue, because his intentional act was firing the rifle, not shooting at the car. (D) is not the best answer, because although there is the possibility that the shooter might have been able to show only gross negligence, there is sufficient evidence to support a finding of malice aforethought and murder.

During a heated argument, a man punched his female coworker in the stomach after learning that she had been awarded "Employee of the Month." Angered, the woman responded by stabbing the man with a letter opener, which severed his aorta and caused his death. The police arrested the woman. After receiving her Miranda rights, she confessed to killing her coworker, but stated that she had not previously held any grudge against him. When asked why she stabbed him, she stated, "He just made me so mad when he said that I was a terrible employee and that he deserved the reward instead of me, and I just lost it when he punched me." The district attorney charged the woman with homicide. If the jury believes the woman's statement, of what crime is she most likely to be found guilty?

Voluntary Manslaughter. The woman would most likely be found guilty of voluntary manslaughter in light of the provoking event. Adequate provocation will reduce a killing to voluntary manslaughter if the defendant was both reasonably provoked and actually provoked. The woman also would have to offer evidence that the insults and taunting, followed by a sudden punch in the stomach, would cause an ordinary person to be provoked, which is likely to be the case. Hence, her statement in conjunction with the facts makes this the best answer. (A) is incorrect. At common law, murder was the unlawful killing of a human being with malice aforethought. Malice aforethought could be established with any one of the following states of mind: (i) intent to kill; (ii) intent to cause serious bodily harm; (iii) the depraved heart killing (a reckless indifference to an unjustifiably high risk to human life); or (iv) the commission of a felony. While at least an intent to cause serious harm may be present here, the provocation will reduce the homicide to voluntary manslaughter. (C) is incorrect because involuntary manslaughter is a killing that results from criminal negligence, which theory these facts do not support. (D) is incorrect because, as discussed above, the woman would likely be guilty of voluntary manslaughter.

A police officer received an anonymous tip that the defendant was at a particular street corner selling drugs. Based solely on the tip, the officer went to the corner, saw the defendant talking with someone, and immediately stopped and frisked the defendant. The officer found an illegal gun as a result of the stop and frisk and arrested the defendant. Prior to trial, the defendant moves to suppress the gun. Should the motion be granted?

Yes, because an anonymous tip, without more, is insufficient to stop and frisk a suspect. The motion to suppress the gun should be granted. Police have the authority to briefly detain a person for investigative purposes even if they lack probable cause to arrest. To make such a stop, police must have a reasonable suspicion supported by articulable facts of criminal activity or involvement in a completed crime. When the source of suspicion of criminal activity is an infor- mant's tip, the tip must be accompanied by indicia of reliability sufficient to make the officer's stop reasonable. Here, the stop was based solely on an informant's tip. Because the tip was anonymous and included nothing more than an accusation that a person standing on a certain street corner was selling drugs, it was not sufficient to justify the search. It needed to include more detail (e.g., predicting incriminating movement) to corroborate the accusation. Thus, (A) is incorrect, and the gun should be suppressed under the exclusionary rule as fruit of the poisonous tree. (B) is incorrect. Under the inevitable discovery exception to the exclusionary rule, evidence may be admissible if the police inevitably would have discovered the evidence whether or not they acted unconstitutionally. However, here it is just speculation that the officer would have had grounds to arrest the defendant; this is not sufficient to apply that exception. (D) is incorrect because Miranda warnings are required for custodial police interrogation, not before a stop and frisk.

A suspect was arrested on a charge of bank robbery. After formal charges had been filed, the suspect was scheduled for a lineup identification procedure. The suspect's attorney was notified of the lineup and arrived at the station prior to the scheduled lineup. He was directed to wait in the hallway outside the lineup room. When the suspect was escorted into the lineup room, the officer acknowledged the attorney but did not motion for him to follow them int the lineup room. Th attorney remained seated int eh hallway. Inside the lines room, the six members of the lineup, including the suspect, stood on one side of the one-way mirror in full view of the witnesses gathered on the other side in the viewing area. The lineup members stepped forward one by one for a closer inspection. After the first two lineup members had been presented, an officer realized that the suspect's attorney was not present and immediately escorted him to the viewing area. The attorney arrived in time to witness the presentation of the suspect and the remaining thermometers of the lineup. Two witnesses then identified the suspect as the robber. At trial, if the defense objects to the introduction of the lineup evidence, should the objection be granted?

Yes, because the right to have counsel present at a post-charge lineup includes the right to have counsel present for the entire lineup. The objection should be granted. A post-charge lineup is a critical stage of the prosecution at which a defendant has the right to counsel. Once the government has initiated adversary judicial criminal proceedings, the presence of counsel is a prerequisite to the conduct of a lineup. This right attaches as soon as the accused is within sight of a potential identification witness. Here, the attorney was not present in the room until after the lineup had commenced. (A) is incorrect because the requirement that counsel be present during a post-charge lineup has never turned on a showing of government bad faith, or even government error. The government has an affirma- tive obligation to ensure counsel's presence at a post-charge lineup. (B) is incorrect because the fact that the attorney was present when the suspect was asked to step forward does not remedy the constitutional violation. The accused is entitled to have counsel present at all times during the lineup procedure when the accused is visible to the witnesses. Because the suspect was visible at all times during the lineup, not just when he stepped forward for closer viewing, he was entitled to counsel at all times during the lineup procedure. (D) is incorrect. It is true that, if the witnesses are unavailable at trial, their "prior identification" testimony may be admissible under the Federal Rules of Evidence. However, the evidence is inadmissible regardless because of a violation of the right to counsel.

A drug smuggler had just returned home after smuggling in a large quantity of cocaine in the false bottom of his suitcase. AS he was about to leave his house again to deliver the cocaine to his contact in the city, a police officer arrived with a trained drug-sniffing dog and asked him if he could come in ands ask him some questions. The smuggler declined but that officer stepped into the doorway, and the dog immediately caught the scent of the cocaine and pulled the officer toward the suitcase in the hallway. Based on the dog's clear indication that the suitcase contained narcotics, the police officer opened the suitcase and found the cocaine. The smuggler was then arrested and the cocaine and suitcase seized. At a pretrial hearing, should the judge grant the smuggler's motion to suppress evidence of the cocaine in the suitcase?

Yes, because the search and seizure required a warrant. The motion should be granted because the search and seizure required a warrant. To have a Fourth Amendment right, a person must have a reasonable expectation of privacy with respect to the place searched or the item seized. In the instant case, the place searched was the defen- dant's home, which clearly is a place in which a person has a reasonable expectation of privacy. Even the entry to a home is within the curtilage and protected against unreasonable searches. Consent to enter was not granted and no other exceptions apply, so evidence of the cocaine should be suppressed. (A) is incorrect because it is too broad of a statement. A warrantless search and seizure of items in one's home may be based on other grounds, such as consent or plain view. (C) is incorrect because the search occurred prior to the arrest and thus was not a search incident to a lawful arrest (even assuming the arrest was lawful). (D) is incorrect. Although it has been held that one does not have a privacy interest in the smell of one's luggage, here the luggage was located in the defendant's home. The fact that the cocaine was still in the defendant's luggage, rather than in some other location within the house, is irrelevant. The search occurred within the defendant's home, which was not permissible under the facts.

The defendant was arrested, given Miranda warnings, and charged with burglary. At the police station, he telephoned his mother and asked her to come to the station to post bail. Instead, his mother immediately called the family attorney. In the meantime, the police had begun questioning the defendant. Although he never told the police to stop the questioning, his answers were at first vague or clearly unresponsive. During the course of the questioning, the family attorney phoned the station and told the police that she had been hired to represent the defendant and would be there in half an hour. The police did not inform the defendant of the attorney's call. Ten minutes later, the defendant admitted to committing the burglary, and signed a statement to that effect prepared by the police. The attorney arrived a few minutes later and advised the defendant to remain silent, but he told her that he had already signed a confession. How should the court rule on the attorney's pretrial motion to exclude the confession as evidence at trial?

he defendant's confession should be admitted because he waived his Fifth Amendment privilege against compelled self-incrimination after receiving Miranda warnings. Miranda v. Arizona requires that a person in custody be informed of his right to remain silent and his right to the presence of an attorney during questioning. A suspect may subsequently waive his rights by making a confession, as long as the waiver was knowing and voluntary. In this case, the defendant received proper Miranda warnings, and there is no indication that he did not understand what his rights were. Although his answers during questioning were initially unresponsive, he never asked for an attorney or indicated that he wished to remain silent, and he voluntarily confessed after a relatively short period of interrogation. Hence, he validly waived his Miranda rights. (A) is incorrect because the police have no duty to inform the defendant that an attorney is attempting to see him. The defendant's ignorance of his attorney's efforts has no bearing on whether he made a knowing waiver of his Miranda rights. (B) is incorrect because the defendant's right to counsel was not violated. Although the defendant does have a separate Sixth Amendment right to counsel under Escobedo v. Illinois because he has already been arrested and charged with the crime, this right would only be violated if the defendant, after being informed of his right to counsel, had requested an attorney or had been prevented from seeing his attorney. Here, he made no request to see an attorney - even when he called his mother - and his attorney was allowed to see him immediately upon her arrival. Thus, he has waived his Sixth Amendment right to counsel. (C) is incorrect even though it is true that the defendant made a voluntary statement. Due process requires that for confessions to be admissible, they must be "voluntary," based on the totality of the circumstances, and here all of the circumstances indicate that the defendant's confession was voluntary. However, even a voluntary confession will be inadmissible if it was obtained in violation of Miranda rights. (D) is therefore a better choice than (C).


Related study sets

Chapter 6: Values, Ethics, and Advocacy

View Set

Chapter 47: Assessment: Endocrine System (Lewis)

View Set

Analytics - Chapter 14 Decision Analysis I

View Set